Difference between revisions of "User:Tohline/Math/EQ Toroidal04"

From VistrailsWiki
Jump to navigation Jump to search
(Created page with '<table border="0" cellpadding="5" align="center"> <tr> <td align="right"> [[Image:LSU_Key.png|25px|link=http://www.vistrails.org/index.php/User:Tohline/Appendix/Equation_templat…')
 
Line 6: Line 6:
</td>  
</td>  
   <td align="right">
   <td align="right">
<math>~Q_\nu^\mu(z)</math>
<math>~(\nu - \mu + 1)P^\mu_{\vu + 1}</math>
   </td>
   </td>
   <td align="center">
   <td align="center">
Line 21: Line 21:
</tr>
</tr>
<tr>
<tr>
   <td align="center" colspan=4>
   <td align="center" colspan="4">
[https://authors.library.caltech.edu/43491/1/Volume%201.pdf A. Erd&eacute;lyi (1953)]:&nbsp; Volume I, &sect;3.7, p. 156, eq. (10)  
[https://books.google.com/books?id=MtU8uP7XMvoC&printsec=frontcover&dq=Abramowitz+and+stegun&hl=en&sa=X&ved=0ahUKEwialra5xNbaAhWKna0KHcLAASAQ6AEILDAA#v=onepage&q=Abramowitz%20and%20stegun&f=false Abramowitz &amp; Stegun (1995)], p. 334, eq. (8.5.3)
   </td>
   </td>
</tr>
</tr>

Revision as of 23:47, 13 June 2018

LSU Key.png

<math>~(\nu - \mu + 1)P^\mu_{\vu + 1}</math>

<math>~=</math>

<math>~ e^{i \mu \pi} ~ (2\pi)^{-\frac{1}{2}} (z^2-1)^{\mu/2} ~\Gamma(\mu + \tfrac{1}{2})~\biggl\{ \int_0^\pi (z - \cos t)^{-\mu - \frac{1}{2}} \cos[(\nu + \tfrac{1}{2})t] ~dt -\cos(\nu\pi) \int_0^\infty (z + \cosh t)^{-\mu - \frac{1}{2}} e^{-(\nu + \frac{1}{2})t} ~dt \biggr\} </math>

Abramowitz & Stegun (1995), p. 334, eq. (8.5.3)

NOTE: Both <math>~P_\nu^\mu</math> and <math>~Q_\nu^\mu</math> satisfy this same recurrence relation.